Using the polar coordinate system to find the volume of a solid with double integrals

burt

Full Member
Joined
Aug 1, 2019
Messages
254
I was given the following problem:
1588515951649.png
I found that \(2\leq r\leq3\) using the equality \(x^2+y^2=r^2\) and then using \(z=x^2+y^2-4\) and \(x^2+y^2=9\). For \(\theta\) I assumed that it ranged from \(0\) to \(\pi\) because of the \(z=0\). But, I realized that on the \(x-y\)-plane the equations trace out full circles and therefore I should use \(0\leq\theta\leq2\pi\). Is this true?

Here is my work:

1588516235063.png
 
Looks good to me. At one point you typed [MATH]\pi[/MATH] instead of [MATH]2\pi[/MATH] as the upper limit, but you fixed it.
 
Top